question types Flashcards

1
Q

If the statements above are true, which one of the following must also be true?

A

must be true/most supported

How well did you know this?
1
Not at all
2
3
4
5
Perfectly
2
Q

which one of the following can be properly inferred from the passage?

A

MBT/most supported

How well did you know this?
1
Not at all
2
3
4
5
Perfectly
3
Q

Larson and Manuela disagree about whether

A

debate/point at issue

How well did you know this?
1
Not at all
2
3
4
5
Perfectly
4
Q

which one of the following is an assumption required by the argument above?

A

assumption (nec)

How well did you know this?
1
Not at all
2
3
4
5
Perfectly
5
Q

which one of the following, if assumed, allows the conclusion above to be properly drawn?

A

justify the conclusion/suff.

How well did you know this?
1
Not at all
2
3
4
5
Perfectly
6
Q

which one of the following, if true, most strengthens the argument?

A

support/strengthen

How well did you know this?
1
Not at all
2
3
4
5
Perfectly
7
Q

which one the following, if true, most strongly support the statement above?

A

support/strengthen

How well did you know this?
1
Not at all
2
3
4
5
Perfectly
8
Q

which one of the following, if true, would most effectively resolve the apparent paradox above?

A

paradox

How well did you know this?
1
Not at all
2
3
4
5
Perfectly
9
Q

which one of the following describes the technique of reasoning used above?

A

method of reasoning

How well did you know this?
1
Not at all
2
3
4
5
Perfectly
10
Q

the reasoning in the ___ argument is flawed because this argument

A

flaw

How well did you know this?
1
Not at all
2
3
4
5
Perfectly
11
Q

which one of the following arguments is most similar in its pattern of reasoning to the argument above?

A

parallel

How well did you know this?
1
Not at all
2
3
4
5
Perfectly
12
Q

the answer to which one of the following questions would contribute most to an evaluation of the argument?

A

evaluate the argument

How well did you know this?
1
Not at all
2
3
4
5
Perfectly
13
Q

if the statements above are true, which one of the following CANNOT be true?

A

cannot be true

How well did you know this?
1
Not at all
2
3
4
5
Perfectly
14
Q

which stimuli don’t contain conclusions?

A

must be true, cannot be true, and resolve the paradox questions

How well did you know this?
1
Not at all
2
3
4
5
Perfectly
15
Q

which stimulus usually don’t have reasoning?

A

weaken or method of reasoning

How well did you know this?
1
Not at all
2
3
4
5
Perfectly
16
Q

If there’s no conclusion and no paradox then it’s a….?

A

must be true

How well did you know this?
1
Not at all
2
3
4
5
Perfectly
17
Q

question types that contain holes

A

weaken (open) and strengthen (close)

How well did you know this?
1
Not at all
2
3
4
5
Perfectly
18
Q

brother/sister pairs

A

method of reasoning and flaw in reasoning

How well did you know this?
1
Not at all
2
3
4
5
Perfectly
19
Q

Difference between method of reasoning and flaw in the reasoning

A

Flaw: stem says there’s a reasoning error in the stimulus

Method: stimulus could be a valid or invalid argument and you have to make a note of that

How well did you know this?
1
Not at all
2
3
4
5
Perfectly
20
Q

one-step extension of method of reasoning?

A

parallel reasoning

21
Q

Task in parallel reasoning question?

A

ID the type of reasoning used and then parallel it

22
Q

Method of reasoning and parallel reasoning both have strong ____ element?

A

must be true!!

23
Q

If the statement above are true, which one of the following conclusion can be properly drawn on the basis of them?

A

must be true

24
Q

Which one of the following is an assumption on which the argument depends?

A

assumption

25
Q

which one of the following, if true, most seriously undermines the argument’s stated conclusion?

A

weaken

26
Q

the argument proceeds by?

A

method of reasoning

27
Q

The pattern of reasoning displayed in the argument above is most closely paralleled by that in which one of the following arguments?

A

parallel of reasoning

28
Q

which one of the following can be logically inferred from the passage?

A

must be true

29
Q

which one of the following, if true, would most strengthen the argument?

A

strengthen

30
Q

which one of the following argumentative strategies is used by the researcher in responding to the manager?

A

method of reasoning

31
Q

which one of the following, if true, argues most strongly against the explanation reported in the passage?

A

weaken

32
Q

which one of the following principles, if established, would do most to support the customer’s position against the advertiser’s response?

A

strengthen

33
Q

which one of the following, if true, most helps to resolve the apparent discrepancy in the information above?

A

paradox

34
Q

if the statements above are true, they provide the most support for which one of the following?

A

must be true

35
Q

Mary uses which one of the following argumentative techniques in countering Paul’s argument?

A

method of reasoning

36
Q

which one of the following contains an error of reasoning that is also contained in the argument above?

A

parallel reasoning

37
Q

the argument is structured to lead to the conclusion that

A

main point

38
Q

which one of the following is most similar in its logical features to the argument above?

A

parallel the reasoning

39
Q

Greg does which one of the following in responding to Sara’s argument?

A

method of reasoning

40
Q

which one of the following is best supported by the passage?

A

must be true

41
Q

which one of the following, if true, most seriously call into question?

A

weaken

42
Q

the judge responds to the political’s argument by doing which one of the following?

A

main point

43
Q

the pattern of reasoning in the argument above is most similar to that in which one of the following?

A

method of reasoning

44
Q

which one of the following, if true, most seriously weaken the argument?

A

weaken

45
Q

which one of the following, if true, most helps to resolve the apparent paradox in the information above?

A

paradox

46
Q

the author’s conclusion depends upon which one of the following?

A

assumption

47
Q

A questionable aspect of the reasoning above is that it

A

Flaw

48
Q

The reasoning in the arg is fallacious because the arg

A

Flaw